based on one statement & two arguments Detailed Explanation And More Example

MOST IMPORTANT verbal reasoning - 2 EXERCISES

Top 10,000+ Verbal Memory Based Exercises

Directions:
Directions: Each question given below consists of a statement, followed by two arguments numbered I and II. You have to decide which of the arguments is a 'strong' argument and which is a 'weak' argument. Give answer
    (a) if only argument I is strong;
    (b) if only argument II is strong;
    (c) if either I or II is strong;
    (d) if neither I nor II is strong and
    (e) if both I and II are strong.

The following question based on statement & arguments topic of verbal reasoning

Questions :
  • Statement:
  • Should agriculture in rural India be mechanized?
  • Argument :
  • I. Yes. It would lead to higher production.
  • II. No. Many villagers would be left unemployed.

(a) if only argument I is strong;

(b) if only argument II is strong;

(c) if either I or II is strong;

(d) if neither I nor II is strong and

e) if both I and II are strong.

The correct answers to the above question in:

Answer: (a)

Clearly, mechanization would speed up the work and increase production. So, the argument I is strong enough.

Argument II is vague because mechanization will only eliminate wasteful employment, not create unemployment.

Practice statement & arguments (based on one statement & two arguments) Online Quiz

Discuss Form

Valid first name is required.
Please enter a valid email address.
Your genuine comment will be useful for all users! Each and every comment will be uploaded to the question after approval.

Read more one statement two arguments questions Based Verbal Reasoning Questions and Answers

Question : 1

  • Statement:
  • Should the educated unemployed youth be paid "unemployment allowance" by the Government?
  • Argument :
  • I. Yes. It will provide them some monetary help to either seek employment or to kickstart some 'self-employment' venture.
  • II. No. It will dampen their urge to do something to earn their livelihood and thus promote idleness among the unemployed youth.

a) if only argument I is strong;

b) if only argument II is strong;

c) if either I or II is strong;

d) if neither I nor II is strong and

e) if both I and II are strong.

Answer: (e)

Young people, who do not get employment due to a large number of applicants in all fields, must surely be given an allowance so that they can support themselves.

So, argument I is valid. However, such allowances would mar the spirit to work, in them and make them idle. So, argument II also holds.

Question : 2

  • Statement:
  • Should all the practising doctors be brought under Government control so that they get a salary from the Government and treat patients free of cost?
  • Argument :
  • I. No. How can any country do such an undemocratic thing?
  • II. Yes. Despite many problems, it will certainly help minimize, if not eradicate, unethical medical practices.

a) if only argument I is strong;

b) if only argument II is strong;

c) if either I or II is strong;

d) if neither I nor II is strong and

e) if both I and II are strong.

Answer: (b)

A doctor treating a patient individually can mislead the patient into wrong and unnecessary treatment for his personal gain. So, argument II holds strong.

Also, a policy beneficial to common people cannot be termed 'undemocratic’. So, I is vague.

Question : 3

  • Statement:
  • Should there be more than one High Courts in each state in India?
  • Argument :
  • I. No. This will be a sheer wastage of taxpayers' money.
  • II. Yes. This will help reduce the backlog of cases pending for a very long time.

a) if only argument I is strong;

b) if only argument II is strong;

c) if either I or II is strong;

d) if neither I nor II is strong and

e) if both I and II are strong.

Answer: (b)

Clearly, an increase in the number of High Courts will surely speed up the work and help to do away with the pending cases. So, argument II holds strong.

In light of this, the expenditure incurred would be ‘utilization', not 'wastage’ of money. So, argument I do not hold.

Directions:
In making decisions about important questions, it is desirable to be able to distinguish between 'strong' arguments and 'weak' arguments. 'Strong' arguments are those which are both important and directly related to the question. 'Weak' arguments are those which are of minor importance and also may not be directly related to the question or may be related to a trivial aspect of the question. Each question below is followed by two arguments numbered I and II. You have to decide which of the arguments is a 'strong' argument and which is a 'weak' argument? Give answer
    (a) if only Argument I is strong
    (b) if only Argument II is strong
    (c) if either I or II is strong
    (d) if neither I nor II is strong
    (e) if both I and II are strong.

Question : 4

  • Statement:
  • Should India make efforts to harness solar energy to fulfill its energy requirement?
  • Argument :
  • I. Yes, most of the energy sources used at present are exhaustible.
  • II. No, harnessing solar energy requires a lot of capital, which India lacks in.

a) if only Argument I is strong

b) if only Argument II is strong

c) if either I or II is strong

d) if neither I nor II is strong

e) if both I and II are strong.

Answer: (a)

Argument I is strong i.e., India should make efforts to harness solar energy to fulfill its energy requirement.

Because most of the energy sources used at present are exhaustible. Argument II is weak.

Question : 5

  • Statement:
  • Should the women be advised not to travel alone at night in view of the increasing incidences of rape and sexual abuse?
  • Argument :
  • I. No, instead the government should take measures to control such incidences.
  • II. Yes, it is difficult to even for the police department to control such cases.

a) if only Argument I is strong

b) if only Argument II is strong

c) if either I or II is strong

d) if neither I nor II is strong

e) if both I and II are strong.

Answer: (d)

Argument I does not establish proper relation with the statement.

Argument II may be an opinion and has not been discussed in the given statement.

Hence, I and II both are weak.

Question : 6

  • Statement:
  • Should it be made compulsory for all the private sector organizations to reserve quota for socially backward classes?
  • Argument :
  • I. No, the private sector should not be governed by the Government rules.
  • II. Yes, private sector organizations should also contribute to the upliftment of socially backward classes.

a) if only Argument I is strong

b) if only Argument II is strong

c) if either I or II is strong

d) if neither I nor II is strong

e) if both I and II are strong.

Answer: (b)

Argument I is not the right way. Thus, Argument I is weak.

Argument II is strong as the upliftment of socially backward classes is necessary.

Recently Added Subject & Categories For All Competitive Exams

New 150+ Percentage Questions For IBPS Clerk Prelims 2024

Free Top Percentage Quants Aptitude based Multiple Choice Questions and Answers Practice Test Series, Quiz PDF & Mock Test for IBPS Clerk Prelims 2024 Exam

04-May-2024 by Careericons

Continue Reading »

Classification Reasoning MCQ For IBPS Clerk Prelims 2024

Most Important 100+ Classification based Verbal Reasoning Ability Multiple choice questions and answers PDF, Free New Mock tests For IBPS Clerk Prelims 2024

03-May-2024 by Careericons

Continue Reading »

Ratio and Proportion Questions with Solutions, IBPS Clerk

New Ratio and Proportion Quants Aptitude based Multiple Choice Questions and Answers Practice Test Series, Quiz & Mock Test for IBPS Clerk Prelims 2024 Exam

02-May-2024 by Careericons

Continue Reading »

Top Analogy Reasoning Ability For IBPS Clerk Prelims 2024

Most Important Analogy based Verbal Reasoning Multiple choice questions and answers practice quiz series PDF, Free New Mock tests For IBPS Clerk Prelims 2024

01-May-2024 by Careericons

Continue Reading »